Last visit was: 10 May 2024, 19:49 It is currently 10 May 2024, 19:49

Close
GMAT Club Daily Prep
Thank you for using the timer - this advanced tool can estimate your performance and suggest more practice questions. We have subscribed you to Daily Prep Questions via email.

Customized
for You

we will pick new questions that match your level based on your Timer History

Track
Your Progress

every week, we’ll send you an estimated GMAT score based on your performance

Practice
Pays

we will pick new questions that match your level based on your Timer History
Not interested in getting valuable practice questions and articles delivered to your email? No problem, unsubscribe here.
Close
Request Expert Reply
Confirm Cancel
SORT BY:
Date
Tags:
Show Tags
Hide Tags
User avatar
Manager
Manager
Joined: 28 Mar 2009
Posts: 54
Own Kudos [?]: 1698 [31]
Given Kudos: 0
Send PM
Most Helpful Reply
Math Expert
Joined: 02 Sep 2009
Posts: 93155
Own Kudos [?]: 622706 [3]
Given Kudos: 81828
Send PM
General Discussion
User avatar
Intern
Intern
Joined: 16 Feb 2009
Posts: 47
Own Kudos [?]: 605 [1]
Given Kudos: 1
Concentration: Real Estate
Schools:Anderson FEMBA
 Q48  V38
Send PM
User avatar
Manager
Manager
Joined: 15 May 2009
Posts: 90
Own Kudos [?]: 71 [1]
Given Kudos: 3
 Q44  V44
Send PM
Re: To decrease the number of crimes in city Y, the city's Police Commissi [#permalink]
1
Kudos
skim wrote:
To decrease the number of crimes in city Y, the city's Police Commissioner proposed taking some police officers from low-crime districts of the city and moving them to high-crime districts of the city. His proposal is based on city Y crime data that show that the number of crimes in any district of the city decreases when additional police officers are moved into that district.
The Police Commissioner's proposal depends on which of the following assumptions?

(A) City X experienced a drastic reduction in crime after implementing a proposal similar to that proposed by the Police Commissioner of city Y.
(B) The severity of crimes committed in any district of the city decreases when additional police officers are moved into that district.
(C) The number of crimes committed in all high-crime districts of city Y is more than triple the number of crimes committed in all low-crime districts of city Y.
(D) There are more low-crime districts than high-crime districts in city Y.
(E) Districts of the city from which police officers are removed do not experience significant crime increases shortly after the removal of those officers.


I would have to go with (E). The goal is to decrease overall crime levels. If low-crime districts will become high-crime districts following the redeployment of cops, then there is no sense moving cops around, because the overall crime level will be unlikely to decrease.

Now a look at the the other answers:
(A) It certainly helps that there is precedence for success in this type of operation, but this particular plan's success does not depend on this assumption. The plan will automatically fall apart if it wasn't successful in city X.
(B) We're talking about the number of crimes, so I don't think severity has much relevance.
(C) Again, using the negation test (or whatever its called), if this wasn't true, the argument doesn't fall apart.
(D) Again, negation test suggests this doesn't necessary make the argument succeed or fail.

(E) is stronger than any other argument available.
avatar
Intern
Intern
Joined: 09 May 2013
Posts: 35
Own Kudos [?]: 25 [0]
Given Kudos: 12
Send PM
Re: To decrease the number of crimes in city Y, the city's Police Commissi [#permalink]
(A) City X experienced a drastic reduction in crime after implementing a proposal similar to that proposed by the Police Commissioner of city Y

if we negate these option choice

City X does not experienced a drastic reduction in crime after implementing a proposal similar to that proposed by the Police Commissioner of city Y

still argument will fall apart because argument clearly states that

"the number of crimes in any district of the city decreases when additional police officers are moved into that district. "

argument clearly uses term "any"
so how can option "A" be wrong..
it also plays a good role of strengthen question.
but after negating it is destroying conclusion as well.
User avatar
Current Student
Joined: 14 Dec 2012
Posts: 580
Own Kudos [?]: 4328 [1]
Given Kudos: 197
Location: India
Concentration: General Management, Operations
GMAT 1: 700 Q50 V34
GPA: 3.6
Send PM
Re: To decrease the number of crimes in city Y, the city's Police Commissi [#permalink]
1
Kudos
WarriorGmat wrote:
(A) City X experienced a drastic reduction in crime after implementing a proposal similar to that proposed by the Police Commissioner of city Y

if we negate these option choice

City X does not experienced a drastic reduction in crime after implementing a proposal similar to that proposed by the Police Commissioner of city Y

still argument will fall apart because argument clearly states that

"the number of crimes in any district of the city decreases when additional police officers are moved into that district. "

argument clearly uses term "any"
so how can option "A" be wrong..
it also plays a good role of strengthen question.
but after negating it is destroying conclusion as well.


if you negate option A:
City X does not experienced a drastic reduction in crime after implementing a proposal similar to that proposed by the Police Commissioner of city Y

CONCLUSION is there will be reduction in crime and not drastic reduction.
thats why this doesnt shatters the conclusion.

hope this helps
avatar
Intern
Intern
Joined: 21 Jun 2013
Posts: 6
Own Kudos [?]: 25 [1]
Given Kudos: 0
Send PM
Re: To decrease the number of crimes in city Y, the city's Police Commissi [#permalink]
1
Kudos
WarriorGmat wrote:
argument clearly uses term "any"
so how can option "A" be wrong..
it also plays a good role of strengthen question.
but after negating it is destroying conclusion as well.


The reason that A is wrong is because it refers to City X, which was never mentioned in the paragraph. The question stem asks us to find the necessary assumption upon which the commissioner's policy for City Y is based. We have no idea about the circumstances of City X's implementation of a "similar" policy; it could be for other reasons that the policy had the intended effect. It really doesn't matter because the circumstances of City X are not necessary for the policy to work in City Y.

The necessary assumption in this case is clearly E, because unless the removal of the police from peaceful areas does not increase the crime rate in those areas, the policy will not work because crime will rise in the originally highly policed areas. In other words, the commissioner is making the necessary assumption that crime free areas will remain crime free, despite losing policing resources to crime filled areas.

Remember, every assumption question refers to a necessary assumption. That means without this assumption the argument will not stand.

A good test for determining necessity is to ask yourself "must this be the case?" Apply this simple method to answer choice A: must it be the case that for City Y's policy to work it also worked for City X? Clearly, this doesn't have to be the case. It is perfectly possible that the policy works in City Y and does not work in City X. Having established that answer choice A is not necessary, we can confidently eliminate it from our choices. If it isn't necessary to the argument, it isn't the correct answer. Apply the same method to E and you will realize that E is necessary for the commissioner's argument to stand.
RC & DI Moderator
Joined: 02 Aug 2009
Status:Math and DI Expert
Posts: 11236
Own Kudos [?]: 32405 [1]
Given Kudos: 301
Send PM
Re: To decrease the number of crimes in city Y, the city's Police Commissi [#permalink]
1
Kudos
Expert Reply
To decrease the number of crimes in city Y, the city’s Police Commissioner proposed taking some police officers from low-crime districts of the city and moving them to high-crime districts of the city. His proposal is based on city Y crime data that show that the number of crimes in any district of the city decreases when additional police officers are moved into that district. The Police Commissioner’s proposal depends on which of the following assumptions?

• City X experienced a drastic reduction in crime after implementing a proposal similar to that proposed by the Police Commissioner of city Y.
closest to be a wrong ans. there is no talk of city X as the data talks of only city Y
• The severity of crimes committed in any district of the city decreases when additional police officers are moved into that district.
scope shift. it talks of severity not number

• The number of crimes committed in all high-crime districts of city Y is more than triple the number of crimes committed in all low-crime districts of city Y.
nothing related to numbers.. triple?

• There are more low-crime districts than high-crime districts in city Y.
out of scope

• Districts of the city from which police officers are removed do not experience significant crime increases shortly after the removal of those officers.
Correct... the assumption is the areas will not be susceptible to more crime if officers are pulled out

ans E
User avatar
Senior Manager
Senior Manager
Joined: 19 Apr 2013
Posts: 477
Own Kudos [?]: 276 [3]
Given Kudos: 537
Concentration: Strategy, Healthcare
Schools: Sloan '18 (A)
GMAT 1: 730 Q48 V41
GPA: 4
Send PM
Re: To decrease the number of crimes in city Y, the city's Police Commissi [#permalink]
3
Kudos
The answer is E. If districts from where officers were removed will suffer from high crime, then overall criminal level of the city won't be affected.
Retired Moderator
Joined: 04 Jun 2013
Status:Getting strong now, I'm so strong now!!!
Affiliations: National Institute of Technology, Durgapur
Posts: 337
Own Kudos [?]: 1902 [2]
Given Kudos: 92
Location: United States (DE)
GPA: 3.32
WE:Information Technology (Health Care)
Send PM
Re: To decrease the number of crimes in city Y, the city's Police Commissi [#permalink]
2
Kudos
A plan will succeed if the solution doesn't inherently create further problems. So the olan is to remove officers from one place to another. Doing so will resuce crime in area that receive additional officers but at the same time won't cause more crime in the area from which offficers are removed.
Option e says the same thing

Posted from my mobile device
Manager
Manager
Joined: 24 Jul 2014
Posts: 71
Own Kudos [?]: 122 [0]
Given Kudos: 39
Location: India
WE:Information Technology (Computer Software)
Send PM
Re: To decrease the number of crimes in city Y, the city's Police Commissi [#permalink]
To decrease the number of crimes in city Y, the city’s Police Commissioner proposed taking some police officers from low-crime districts of the city and moving them to high-crime districts of the city. His proposal is based on city Y crime data that show that the number of crimes in any district of the city decreases when additional police officers are moved into that district. The Police Commissioner’s proposal depends on which of the following assumptions?

Conclusion lies in the last sentence : the number of crimes in any district of the city decreases when additional police officers are moved into that district
So , to decrease the number of crime they are not just adding more officers to a district. They simply borrow additional officers from other district where crime is low
But this methods decreases the number of police officers in those districts and we have to make sure this does not anyways impact this low crime district

Lets look at (E)
E. Districts of the city from which police officers are removed do not experience significant crime increases shortly after the removal of those officers.

- It says the low crime cities from where officers are removed do not experience crime increase. So , their conditions are stable and not going deteriorate in a while
So the proposal is going to work
Hence Answer is (E)
User avatar
Jamboree GMAT Instructor
Joined: 15 Jul 2015
Status:GMAT Expert
Affiliations: Jamboree Education Pvt Ltd
Posts: 252
Own Kudos [?]: 655 [0]
Given Kudos: 1
Location: India
Send PM
Re: To decrease the number of crimes in city Y, the city's Police Commissi [#permalink]
The plan is to remove police officers from the low crime districts to the high crime districts. Besides it is also stated in the argument that when additional officers are placed in a high crime district the number of crime reduces. The very proposal is based on the assumption the the low crime districts will continue to be low crime districts even after the number of officers decrease. The paraphrase of this statement is in "E". If we work on elimination the "A" states the consequence, "B" talks about severity and not the number of crimes, "C" again gives a consequence and " D" is off context.
Verbal Forum Moderator
Joined: 08 Dec 2013
Status:Greatness begins beyond your comfort zone
Posts: 2100
Own Kudos [?]: 8836 [0]
Given Kudos: 171
Location: India
Concentration: General Management, Strategy
GPA: 3.2
WE:Information Technology (Consulting)
Send PM
Re: To decrease the number of crimes in city Y, the city's Police Commissi [#permalink]
souvik101990 wrote:
To decrease the number of crimes in city Y, the city’s Police Commissioner proposed taking some police officers from low-crime districts of the city and moving them to high-crime districts of the city. His proposal is based on city Y crime data that show that the number of crimes in any district of the city decreases when additional police officers are moved into that district. The Police Commissioner’s proposal depends on which of the following assumptions?

A. City X experienced a drastic reduction in crime after implementing a proposal similar to that proposed by the Police Commissioner of city Y.

B. The severity of crimes committed in any district of the city decreases when additional police officers are moved into that district.

C. The number of crimes committed in all high-crime districts of city Y is more than triple the number of crimes committed in all low-crime districts of city Y.

D. There are more low-crime districts than high-crime districts in city Y.

E. Districts of the city from which police officers are removed do not experience significant crime increases shortly after the removal of those officers.



Type- Assumption
Boil it down -Transferring some police officers from low-crime to high crime district will read to decrease in number of crimes in the city Y
"To decrease the number of crimes in city Y, the city’s Police Commissioner proposed taking some police officers from low-crime districts of the city and moving them to high-crime districts of the city "

What if low-crime districts experience a surge in the number of crimes post reduction in police personnel .
If the above scenario is true , then the commissioners' proposal to reduce the number of crimes by transferring police officers will be counterproductive .

Total crime in city Y = number of crimes in district D1(low-crime) .......+ number of crimes in district D2(high-crime)

E. Districts of the city from which police officers are removed do not experience significant crime increases shortly after the removal of those officers. - The argument falls apart on negation of this statement

Answer E
Intern
Intern
Joined: 07 Dec 2016
Posts: 34
Own Kudos [?]: 25 [0]
Given Kudos: 74
Concentration: Strategy, International Business
Send PM
Re: To decrease the number of crimes in city Y, the city's Police Commissi [#permalink]
Answer : E

• City X experienced a drastic reduction in crime after implementing a proposal similar to that proposed by the Police Commissioner of city Y. (This option can be utilized as a strengthner

• The severity of crimes committed in any district of the city decreases when additional police officers are moved into that district.(The paragraph discussed about number of crimes to number of police and not the severity of crimes)

• The number of crimes committed in all high-crime districts of city Y is more than triple the number of crimes committed in all low-crime districts of city Y.(The para does not provides comparison of the high and low crime areas)

• There are more low-crime districts than high-crime districts in city Y.(The para does not provides comparison of the high and low crime areas)

• Districts of the city from which police officers are removed do not experience significant crime increases shortly after the removal of those officers.(This assumption is a permanent solution to reduce crime in City Y
Senior Manager
Senior Manager
Joined: 11 Mar 2014
Posts: 341
Own Kudos [?]: 111 [1]
Given Kudos: 8
Send PM
Re: To decrease the number of crimes in city Y, the city's Police Commissi [#permalink]
1
Kudos
Hope your preparation is going well.
To solve such questions, the first step should be to read the question.
1. The question gives you clarity about the task. In this case, it is asked to identify the assumption. The role of the assumption is to the bridge the gap that exists between premise and conclusion.
2. Next, the argument should be understood.
While reading the argument, the following pattern can be observed
Planning Pattern
Premise – There is a plan to solve the problem.
Conclusion –Do the plan!!
Assumption- there is no problem with the plan
Weakening- Identify problems with the plan
Strengthening- provide solutions to the problems
Let us see the application
The commissioner intends to decrease the number of crimes by moving the police officers from low crime to high crime districts. The proposal of the commissioner is based on the data of city Y which shows that employing additional forces helps to reduce crimes.
.However, the data do not mention anything about the effect on the districts from which the police officers were removed. This is the flaw/gap/loophole present in the argument .The role of the assumption is to bridge this gap. So, in this case it is assumed that movement of police officers will not have any negative effect on the low crime districts and the plan is going to work.
3. The next step is to predict the answer
The plan is going to work or there is no problem with the plan.
4. Eliminate the incorrect choices and reach to the right one
A –This choice is out of scope. The argument nowhere discusses about City X. This analogy is not bridging any gap.
B – The commissioner’s plan intends to reduce number of crimes in city Y .The severity of the crimes is not discussed in the argument. This choice can also be eliminated as it is out of scope.
C-The comparison drawn between high and low crime areas of the city does not influence the commissioner’s decision.
D- This answer choice has no relevance in the discussion and hence is out of scope.
E –This answer choice matches with the predicted answer.

Hope it helps. Please get in touch if you require further help.
Keep practicing.
All the best!!
PythaGURUS Faculty Team
Director
Director
Joined: 20 Sep 2016
Posts: 559
Own Kudos [?]: 937 [0]
Given Kudos: 632
Location: India
Concentration: Strategy, Operations
GPA: 3.6
WE:Operations (Consumer Products)
Send PM
Re: To decrease the number of crimes in city Y, the city's Police Commissi [#permalink]
GMATNinja mikemcgarry daagh chetan2u DmitryFarber

Though I selected E, I still have a doubt.
The goal - decrease in the number of crimes in city Y.
Plan - move police from city X to city Y

Option E addresses the concern regarding the situation in X when police are removed from X.

Even if we negate this , there is still a possibility that there will be a decrease in crimes in CITY Y...

For eg:
I have 10 candies. I want 2 more.
Joe has whatever amount (greater than or equal 2) of candies. I'll take 2 from Joe..
Now why should I care if Joe is left with any candies... My ONLY goal is increase in no. Of candies I possess.

Similarly, the goal here is DECREASE IN NO. of crimes in CITY Y ...even if there crime increases in city X , we may still have a possibility of our goal reaching.

Posted from my mobile device
Director
Director
Joined: 24 Oct 2016
Posts: 581
Own Kudos [?]: 1327 [0]
Given Kudos: 143
GMAT 1: 670 Q46 V36
GMAT 2: 690 Q47 V38
GMAT 3: 690 Q48 V37
GMAT 4: 710 Q49 V38 (Online)
Send PM
Re: To decrease the number of crimes in city Y, the city's Police Commissi [#permalink]
skim wrote:
To decrease the number of crimes in city Y, the city's Police Commissioner proposed taking some police officers from low-crime districts of the city and moving them to high-crime districts of the city. His proposal is based on city Y crime data that show that the number of crimes in any district of the city decreases when additional police officers are moved into that district.

The Police Commissioner's proposal depends on which of the following assumptions?

This question is part of the GMAT Club Critical Reasoning : Assumption" Revision Project.



(A) City X experienced a drastic reduction in crime after implementing a proposal similar to that proposed by the Police Commissioner of city Y. - Out of Scope

(B) The severity of crimes committed in any district of the city decreases when additional police officers are moved into that district. - Out of Scope

(C) The number of crimes committed in all high-crime districts of city Y is more than triple the number of crimes committed in all low-crime districts of city Y. - Out of Scope

(D) There are more low-crime districts than high-crime districts in city Y. - Out of Scope

(E) Districts of the city from which police officers are removed do not experience significant crime increases shortly after the removal of those officers. - Correct
VP
VP
Joined: 14 Feb 2017
Posts: 1114
Own Kudos [?]: 2167 [0]
Given Kudos: 368
Location: Australia
Concentration: Technology, Strategy
GMAT 1: 560 Q41 V26
GMAT 2: 550 Q43 V23
GMAT 3: 650 Q47 V33
GMAT 4: 650 Q44 V36
GMAT 5: 600 Q38 V35
GMAT 6: 710 Q47 V41
WE:Management Consulting (Consulting)
Send PM
Re: To decrease the number of crimes in city Y, the city's Police Commissi [#permalink]
The proposal is to take some police officers from low-crime districts and insert them into high-crime districts. The proposal is intended to decrease the total number of crimes in City Y

What does the proposal, if successful, assume?
A - what is true for others may not be true for Y. Incorrect
B - severity and number of crimes are mutually exclusive. Incorrect.
C - Even if it were triple, if low-crime districts are now unpoliced that may mean crime increases 10x, thus increasing the total number of crimes in the city overall. C is incorrect
D - we are concerned with the number of crimes. We can have 10 low crime districts each with 1 crime each or 5 districts with 2 crimes each, the total number of crimes is the same.
E - is correct. The commissioner must assume that low-crime districts' total number of crimes WONT increase DRAMATICALLY because if it did then the total crime would be greater than before.
User avatar
Non-Human User
Joined: 01 Oct 2013
Posts: 17291
Own Kudos [?]: 849 [0]
Given Kudos: 0
Send PM
Re: To decrease the number of crimes in city Y, the city's Police Commissi [#permalink]
Hello from the GMAT Club VerbalBot!

Thanks to another GMAT Club member, I have just discovered this valuable topic, yet it had no discussion for over a year. I am now bumping it up - doing my job. I think you may find it valuable (esp those replies with Kudos).

Want to see all other topics I dig out? Follow me (click follow button on profile). You will receive a summary of all topics I bump in your profile area as well as via email.
GMAT Club Bot
Re: To decrease the number of crimes in city Y, the city's Police Commissi [#permalink]
Moderators:
GMAT Club Verbal Expert
6925 posts
GMAT Club Verbal Expert
238 posts
CR Forum Moderator
832 posts

Powered by phpBB © phpBB Group | Emoji artwork provided by EmojiOne